Đến nội dung

Hình ảnh

Khoảng trời dành cho sự sáng tạo


  • Please log in to reply
Chủ đề này có 27 trả lời

#1
Sk8ter-boi

Sk8ter-boi

    (~.~)rubby(^.^)

  • Thành viên
  • 427 Bài viết
cho x;y;z>0 CMR
$\dfrac{3x}{(x+y+z)^2+9y^2} + \dfrac{3y}{(x+y+z)^2+9z^2} + \dfrac{3z}{(x+y+z)^2+9x^2} \geq \dfrac{3}{2(x+y+z)} $

xuất sứ :sáng tác hehe(khoe chút cho zui :Leftrightarrow)

@TIG Messi: Đề sai hoặc là em viết thừa thì phải :in, nếu vậy thì bỏ lun 3 ở tử số đi:
Hình đã gửi


--------
vâng chính thế , xin phép nói thêm
BDT với n số ko âm
$ \sum\limits_{i=1}^{n} \dfrac{x_i}{( \sum\limits_{i=1}^{n}x_1)^2+n^2x_{i+1}^2 } \geq \dfrac{1}{2( \sum\limits_{i=1}^{n}x_i) } $
đc gọi tạm là BDT hannah :Leftrightarrow

Bài viết đã được chỉnh sửa nội dung bởi bboy114crew: 16-07-2011 - 09:17

i love 9C -- i luv u :x .... we'll never fall apart , but shine forever

9C - HN ams

#2
dtdong91

dtdong91

    Tiến sĩ diễn đàn toán

  • Hiệp sỹ
  • 1791 Bài viết
Đúng là rất thú vị
Chuẩn hóa x+y+z=3 ta được
$ \sum \dfrac{x}{9+9y^{2}} \geq \dfrac{1}{6}$
Lưu ý $ \dfrac{9x}{9+9y^{2}}=x (1-\dfrac{9y^{2}}{9+9y^{2}}) \geq x (1-\dfrac{y}{2})$ là được

Bài viết đã được chỉnh sửa nội dung bởi bboy114crew: 16-07-2011 - 09:18

12A1-THPT PHAN BỘI CHÂU-TP VINH-NGHỆ AN

SẼ LUÔN LUÔN Ở BÊN BẠN

#3
Sk8ter-boi

Sk8ter-boi

    (~.~)rubby(^.^)

  • Thành viên
  • 427 Bài viết
vâng ; đó là 1 lời giải rất hay vì có sử dụng pp chuẩn hóa và Am-Gm ngược dấu ..
như các bạn đã biết ; việc sáng tạo ra các bdt là 1 nhu cầu thiết yếu và cũng khá thú vị ; nhất là đối với những học sinh muốn đi sâu vào BDT hay những học sinh phải làm chuyên đề...(như tui nah` :))
quay lại bài toán trên ; con đường đi đến nó khá đơn giản
nếu ta đặt $x= \dfrac{3a}{a+b+c} ;y= \dfrac{3b}{a+b+c} ;z= \dfrac{3c}{a+b+c}$
chúng ta sẽ có BDT tương đương
a;b;c >0 ; a+b+c=3
$\sum \dfrac{a}{1+b^2} \geq \dfrac{3}{2}$
đây là 1 BDT khá quen thuộc có thể giải bằng Am-Gm ngược dấu

Với ý tưởng tương tự như vậy ; ta có thể lập ra 1 số BDT như sau:
với các biến tham gia dương x;y;z;t ta luôn có :
$1) \sum \dfrac{4x+y+z}{(x+y+z)^2+9y^2} \geq \dfrac{3}{x+y+z}$
$2) \sum \dfrac{x^2}{x(x+y+z)^2+18y^3} \geq \dfrac{1}{3(x+y+z)}$
$3) \sum\limits_{cyc(x;y;z;t)}\dfrac{x}{(x+y+z+t)^3+64y^2z} \geq \dfrac{1}{2(x+y+z+t)^2} $

giờ hãy thử làm 1 bài khác
cho a;b;c là các số thực dương ; cmr:
$ \dfrac{8a^3}{b^3} + \dfrac{b^3}{216c^3} + \dfrac{27c^3}{a^3} \geq \dfrac{2 \sqrt{a} }{ b\sqrt{6c} }\sqrt{ab+18c^2} + \dfrac{ \sqrt{b} }{6c \sqrt{a}}\sqrt{3bc+2a^2} + \dfrac{3 \sqrt{c} }{a\sqrt{b} }\sqrt{b^2+12ac} $

N.xét:bài này lộ hơn các bài trước; nhưng ở đây tôi muốn nhấn mạnh dạng TÍCH....

Bài viết đã được chỉnh sửa nội dung bởi Hannah Montana: 21-01-2007 - 10:23

i love 9C -- i luv u :x .... we'll never fall apart , but shine forever

9C - HN ams

#4
Sk8ter-boi

Sk8ter-boi

    (~.~)rubby(^.^)

  • Thành viên
  • 427 Bài viết
1 BĐT mới cho ra lò từ ....hum qua :D
CMR với a,b,c >1/3 ta luôn có
$\sum \sqrt[4]{a}+ \sum 2 \sqrt[4]{a+3b} \geq \sum \sqrt[4]{1-\dfrac{2b}{a+2b} } +2\sum \sqrt[4]{ 1+\dfrac{b}{a+2b} } $

Bài viết đã được chỉnh sửa nội dung bởi bboy114crew: 16-07-2011 - 09:18

i love 9C -- i luv u :x .... we'll never fall apart , but shine forever

9C - HN ams

#5
TIG Messi

TIG Messi

    ^_^ Need + Enough = Success ^_^

  • Thành viên
  • 368 Bài viết

1 BĐT mới cho ra lò từ ....hum qua :D
CMR với a,b,c >1/3 ta luôn có
$\sum \sqrt[4]{a}+ \sum 2 \sqrt[4]{a+3b} \geq \sum \sqrt[4]{1-\dfrac{2b}{a+2b} } +2\sum \sqrt[4]{ 1+\dfrac{b}{a+2b} } $

Bài này hiển nhiên thì phải :D Có chú ý chút là a,b,c :D 1/3 thì mới có dấu bằng :D
BĐT tương đương với:
Hình đã gửi
Hình đã gửi
Điều này đúng theo giả thiết của bài toán

Bài viết đã được chỉnh sửa nội dung bởi bboy114crew: 16-07-2011 - 09:19


#6
Sk8ter-boi

Sk8ter-boi

    (~.~)rubby(^.^)

  • Thành viên
  • 427 Bài viết
ấy chà; xin lỗi ; đk là a+b+c=1 ...

p/s: đặt theo kiểu anh ducpbc sai ở phân thức thứ 3 của VT nên ko trở thành đối xứng đc ; nhân đây xin nêu thêm 1 bài khá lỏng
a_i>0.CM
$ \sum\limits_{i=1}^{n} \dfrac{i \sqrt{i} }{ \sqrt{a_i} } \geq \dfrac{2n \sqrt{n} }{ \sqrt{2(n+1)( \sum\limits_{i=1}^{n} a_i)} } $

Bài viết đã được chỉnh sửa nội dung bởi bboy114crew: 16-07-2011 - 09:26

i love 9C -- i luv u :x .... we'll never fall apart , but shine forever

9C - HN ams

#7
Sk8ter-boi

Sk8ter-boi

    (~.~)rubby(^.^)

  • Thành viên
  • 427 Bài viết
hâm nóng bầu ko khí 1 chút với 1 BDT tiếp theo trong cđề của em(khoe lần nữa :))
cho x;y;z>0 cmr
$\sum \dfrac{x}{2x^2+yz} \geq \dfrac{9xyz}{(xy+yz+xz)^2}$

Bài viết đã được chỉnh sửa nội dung bởi bboy114crew: 16-07-2011 - 09:27

i love 9C -- i luv u :x .... we'll never fall apart , but shine forever

9C - HN ams

#8
Sk8ter-boi

Sk8ter-boi

    (~.~)rubby(^.^)

  • Thành viên
  • 427 Bài viết

cho a;b;c là các số thực dương ; cmr:
$ \dfrac{8a^3}{b^3} + \dfrac{b^3}{216c^3} + \dfrac{27c^3}{a^3} \geq \dfrac{2 \sqrt{a} }{ b\sqrt{6c} }\sqrt{ab+18c^2} + \dfrac{ \sqrt{b} }{6c \sqrt{a}}\sqrt{3bc+2a^2} + \dfrac{3 \sqrt{c} }{a\sqrt{b} }\sqrt{b^2+12ac} $

N.xét:bài này lộ hơn các bài trước; nhưng ở đây tôi muốn nhấn mạnh dạng TÍCH....


bài trên lời giải khá đơn giản
trước tiên ta đặt $\dfrac{2a}{b}=x ; \dfrac{b}{6c}=y ; \dfrac{3c}{a}=z $
ta có BDT tương đương với
x;y;z>0 và xyz=2
CMR: $x^3+y^3+z^3 \geq x\sqrt{y+z} +y\sqrt{x+z} +z\sqrt{x+y}$
mặt khác ta có $x^3+y^3+z^3 \geq \dfrac{(x+y+z)(x^2+y^2+z^2)}{3}= \dfrac{[(x+y)+(y+z)+(z+x)](x^2+y^2+z^2)}{6} \geq \dfrac{(x\sqrt{y+z} +y\sqrt{x+z} +z\sqrt{x+y})^2 }{6} $
bài toán trở thành CM
$x\sqrt{y+z} +y\sqrt{x+z} +z\sqrt{x+y} \geq 6$
xyz=2 nên bài toán trên là khá dễ dàng ...

chú ý rằng ; ngoài việc lao đầu vào chứng minh 1 bài toán ; ta nên có cái nhìn tổng quan về bài toán đó trước , để thấy đc sự quan hệ giữa các bài toán với nhau .
ở đây cũng nêu thêm ; đôi khi , từ 1 BDT cho trước ; ta cũng có thể tạo ra 1 BDT mới , và điều quan trọng là che giấu đi ĐK của BDT cũ .... , như ở trên , chúng ta đã che dấu đi đc đk abc=2

tiếp tục với 1 BĐt , tư tưởng đặt ẩn phụ khá hay .... và khó :delta
[Gabriel Dospinescu]
cho a,b,c>0 a+b+c=6
CMR
$\sqrt{a+1} + \sqrt{b+1} + \sqrt{c+1} \geq \sqrt{15+ab+bc+ac} $

Bài viết đã được chỉnh sửa nội dung bởi bboy114crew: 16-07-2011 - 09:27

i love 9C -- i luv u :x .... we'll never fall apart , but shine forever

9C - HN ams

#9
dtdong91

dtdong91

    Tiến sĩ diễn đàn toán

  • Hiệp sỹ
  • 1791 Bài viết

tiếp tục với 1 BĐt , tư tưởng đặt ẩn phụ khá hay .... và khó :delta
[Gabriel Dospinescu]
cho a,b,c>0 a+b+c=6
CMR
$\sqrt{a+1} + \sqrt{b+1} + \sqrt{c+1} \geq \sqrt{15+ab+bc+ac} $

Hừm đặt a+1=x,...
=> x+y+z=9
và $ \sum \sqrt{x} \geq \sqrt{xy+yz+zx}$
Cái này bình phương lên thôi

Bài viết đã được chỉnh sửa nội dung bởi bboy114crew: 16-07-2011 - 09:28

12A1-THPT PHAN BỘI CHÂU-TP VINH-NGHỆ AN

SẼ LUÔN LUÔN Ở BÊN BẠN

#10
Sk8ter-boi

Sk8ter-boi

    (~.~)rubby(^.^)

  • Thành viên
  • 427 Bài viết
xin hỏi anh dtdong , sau khi bình phương lên rồi làm tiếp thế nào nhỉ , anh mới chỉ đưa ra 1 BDt tương đương chứ chưa có 1 sự đánh giá cụ thể nào :sqrt{a}

đã khá lâu rồi ; xin đưa ra lời giải của bài tập sau:

$\sum\limits_{i=1}^{n} \dfrac{i \sqrt{i} }{ \sqrt{a_i} } \geq \dfrac{2n\sqrt{n} }{ \sqrt{2(n+1)( \sum\limits_{i=1}^{n}a_i } }$

$ \sum\limits_{i=1}^{n} \dfrac{i \sqrt{i} }{ \sqrt{a_i} }= \sum\limits_{i=1}^{n} \dfrac{1}{ \sqrt{ i\dfrac{a_i}{i} } } \geq \dfrac{n^2}{ \sum\limits_{i=1}^{n}i \sqrt{\dfrac{a_i}{i} } } \geq \dfrac{n^2}{ \dfrac{n(n+1)}{2} \sqrt{ \dfrac{ \sum\limits_{i=1}^{n} \dfrac{a_i}{i} }{ \dfrac{n(n+1)}{2} } } } =\dfrac{2n\sqrt{n} }{ \sqrt{2(n+1)( \sum\limits_{i=1}^{n}a_i )} } $-->dpcm

điều nhấn mạnh ở đây , các bạn vẫn hay sử dụng BDT cauchy dạng lũy thừa
$ \dfrac{ \sum\limits_{i=1}^{n}a_i^m }{n} \geq ( \dfrac{ \sum\limits_{i=1}^{n}a_i }{n})^m $
nhưng lại hay quên BDT song hành với BDT trên là
$\dfrac{ \sum\limits_{i=1}^{n} \sqrt[n]{a_i} }{m} \leq \sqrt[n]{ \dfrac{ \sum\limits_{i=1}^{n}a_i }{m} } $

tiếp theo hãy thử sức với 1 BDt khá kỳ dị nhưng con đường đi đến nó thì khá thú vị
với a,b,c>0 cmr
$\dfrac{ \sqrt[4]{a} }{ \sqrt{ \sqrt{a^9}+8 \sqrt[4]{b^9.c^9} } } +\dfrac{ \sqrt[4]{b} }{ \sqrt{ \sqrt{b^9}+8 \sqrt[4]{a^9.c^9} } }+\dfrac{ \sqrt[4]{c} }{ \sqrt{ \sqrt{c^9}+8 \sqrt[4]{a^9.b^9} } } \geq 1$

Bài viết đã được chỉnh sửa nội dung bởi bboy114crew: 16-07-2011 - 09:29

i love 9C -- i luv u :x .... we'll never fall apart , but shine forever

9C - HN ams

#11
dtdong91

dtdong91

    Tiến sĩ diễn đàn toán

  • Hiệp sỹ
  • 1791 Bài viết
Hì có gì đâu em đến đó rùi nhân 3 vào Vp và $ \sqrt{x+y+z}$ vào VT tùi bình phương lên và xài Cauchy-Schwarz đưa về c/m 1 BDt quen biết sau
$ \sum x^2+\sum 2x\sqrt{yz} \geq \sum 3xy $
c/m cái này có thể dùng dồn biến hoặc S.O.S một phát :sqrt{a}

Bài viết đã được chỉnh sửa nội dung bởi bboy114crew: 16-07-2011 - 09:29

12A1-THPT PHAN BỘI CHÂU-TP VINH-NGHỆ AN

SẼ LUÔN LUÔN Ở BÊN BẠN

#12
sieuthamtu_sieudaochit

sieuthamtu_sieudaochit

    Binh nhất

  • Thành viên
  • 30 Bài viết
Mình cũng có sáng tác một số bài bất đẳng thức, các bạn xem thử nhé.

1. Cho $x,y,z$ là các số thưc không âm thỏa $xyz=1$.CMR
$\dfrac{x^4-x}{x^2+y+z}+\dfrac{y^4-y}{y^2+z+x}+\dfrac{z^4-z}{z^2+x+y}\geq 0$
Dấu bằng xảy ra khi nào ?

2. Cho các số thực dương thỏa $a^2+b^2+c^2=9$
Tìm GTLN của

$P=\sqrt{ab(6-bc)(6-ca)}+\sqrt{bc(6-ca)(6-ab)}+\sqrt{ca(6-ab)(6-bc)}$
P/s. Quên mất bài này giải sau roài.

3. Cho các số dương thỏa $ab+bc+ca \le 24abc$.Chứng minh rằng

$\dfrac{a}{\sqrt{a^2+b}}+\dfrac{b}{\sqrt{b^2+c}}+\dfrac{c}{\sqrt{c^2+a}} \ge 1$

Dấu bằng xảy ra khi nào ?

4. Cho $a,b,c$ là các số dương thỏa $abc=1$. Chứng minh rằng

$\dfrac{a^2}{\sqrt{a^4+4(b+c)}}+\dfrac{b^2}{\sqrt{c^4+4(a+b)}}+\dfrac{c^2}{\sqrt{c^4+4(a+b)}}\geq 1$

5. Cho các số thức dương thỏa $a^3+b^3+c^3=3$. CMR

$\dfrac{abc}{a^4+b+c}+\dfrac{abc}{b^4+c+a}+\dfrac{abc}{c^4+a+b}\leq \dfrac{a+b+c}{a^2+b^2+c^2}$

6. Cho $x,y,z$ là các số dương thỏa $x^2+y^2+z^2 \ge x+y+z$. Chứng minh rằng

$\dfrac{x^3-x^2}{x^3+y^2+z^2}+\dfrac{y^3-y^2}{y^3+z^2+x^2}+\dfrac{z^3-z^2}{z^3+x^2+y^2} \ge 0$

7. Cho $x,y,z$ là các số dương thỏa $xyz=1$. Chừng minh rằng

$\dfrac{x^5}{x^7(y^2+z^2)+1}+\dfrac{y^5}{y^7(z^2+x^2)+1}+\dfrac{z^5}{z^7(x^2+y^2)+1} \le 1$

P/s. Bài này là do em chế lại thoai.

8. Cho $a,b,c$ là các số dương thỏa $a+b+c=1$. Chứng minh rằng:

$(\sqrt{\dfrac{a^2+b^2}{2}}+1)(\sqrt{\dfrac{b^2+c^2}{2}}+1)(\sqrt{\dfrac{c^2+a^2}{2}}+1) > (8abc+1)^3$

9. Cho $a,b,c$ là các số dương thỏa $abc=1$. Chứng minh rằng

$a+b+c \ge\dfrac{1}{a}(2-c)+\dfrac{1}{b}(2-a) +\dfrac{1}{c}(2-b)$

10. Cho $a,b,c$ là các số dương thỏa

$\dfrac{ab}{c}+\dfrac{bc}{a}+\dfrac{ca}{b}=6$.

Tìm GTNN của

$P=\dfrac{1}{1+a}+\dfrac{1}{1+b}+\dfrac{1}{1+c}$

Bài viết đã được chỉnh sửa nội dung bởi bboy114crew: 16-07-2011 - 10:59

[TEX] [/TEX]
Cái này là gì thế nhỉ

#13
Te.B

Te.B

    Once [I]MC-ers ~ 4ever [I]MC-ers

  • Thành viên
  • 104 Bài viết

9. Cho $a,b,c$ là các số dương thỏa $abc=1$. Chứng minh rằng

$a+b+c \ge\dfrac{1}{a}(2-c)+\dfrac{1}{b}(2-a) +\dfrac{1}{c}(2-b)$

Em chém bài 9 ( dễ nhất)
Do a,b,c là các số nguyên dương và $ abc=1 $ nên $ \Rightarrow a = \dfrac{1}{bc} \Rightarrow \dfrac{1}{a}=bc $
Tương tự ta cũng có $ \dfrac{1}{b}=ac; \dfrac{1}{c}=ab $
Do đó bdt phải chứng minh tương đương:
$ a+b+c \geq bc(2-c) +ac(2-a)+ ab(2-b)
\Leftrightarrow a+b+c \geq 2bc - b{c}^{2} + 2ac - {a}^{2}c + 2ab - a{b}^{2}
\Leftrightarrow a({b}^{2}-2b+1) + b({c}^{2}-2c+1) + c({a}^{2}-2a+1) \geq 0
\Leftrightarrow a{(b-1)}^{2}+b{(c-1)}^{2}+c{(a-1)}^{2} \geq 0 $ (luôn đúng với mọi a,b,c thỏa mãn giả thiết)
Dấu "=" xảy ra <=> a=b=c=1

Bài viết đã được chỉnh sửa nội dung bởi bboy114crew: 16-07-2011 - 09:22

ĐI THI TA VỐN KHÔNG HAM ;))
NHƯNG VÌ CÓ GIẢI NÊN LÀM CHO VUI ;))
T/G: CRAZY FAN OF NO-EXAM CLUB =))


#14
khanhtm

khanhtm

    Super Monkey

  • Thành viên
  • 63 Bài viết

Mình cũng có sáng tác một số bài bất đẳng thức, các bạn xem thử nhé.

10. Cho $a,b,c$ là các số dương thỏa

$\dfrac{ab}{c}+\dfrac{bc}{a}+\dfrac{ca}{b}=6$.

Tìm GTNN của

$P=\dfrac{1}{1+a}+\dfrac{1}{1+b}+\dfrac{1}{1+c}$

Mấy bài của ông trông thì khủng nhưng hình như ko khó.
Bài này, $\dfrac{ab}{c}+\dfrac{bc}{a}+\dfrac{ca}{b}=6 \ge a+b+c$
Suy ra, $P=\dfrac{1}{1+a}+\dfrac{1}{1+b}+\dfrac{1}{1+c} \ge \dfrac{9}{3+a+b+c} \ge 1$
Vậy min P = 1 khi a=b=c=2

Bài viết đã được chỉnh sửa nội dung bởi bboy114crew: 16-07-2011 - 09:30


#15
AvidAbel_9x09

AvidAbel_9x09

    Hạ sĩ

  • Thành viên
  • 50 Bài viết
Em cũng có một bài(hình như đã đăng rùi nhưng ko thấy ai trả lời :wub: )
"Cho a,b,c là ba số ko âm thỏa mãn $ a+b+c=3$. CMR:
$ A=a^{4}/(a-b)(a-c)+b^{4}/(b-c)(b-a)+c^{4}/(c-a)(c-b) <27/4$"
đây là bài do em tự chế ko bít có vấn đề ji ko?nhưng em thử làm rùi! Chắc là đúng :wub: . Xin đc chỉ giáo thếm

#16
terenceTAO

terenceTAO

    mathematics...

  • Thành viên
  • 197 Bài viết
mình thử chém bài 1xem sao ap dụng bất đẳng thức cauchy-schwarz ta có $\(z+y+1)(x^2+y+z)>=(x+y+z)^2$

$\dfrac{x^4-x}{x^2+y+z}$ >=$\dfrac{(x^4-x)(y+z+1)}{(x+y+z)^2}$
tương tư với 3 phân thức còn lại
sau đó lam đơn giản thôi

Bài viết đã được chỉnh sửa nội dung bởi bboy114crew: 16-07-2011 - 09:32

Stay hungry,stay foolish


#17
andymurray44

andymurray44

    Trung sĩ

  • Thành viên
  • 153 Bài viết

mình thử chém bài 1xem sao ap dụng bất đẳng thức cauchy-schwarz ta có $\(z+y+1)(x^2+y+z)>=(x+y+z)^2$

$\dfrac{x^4-x}{x^2+y+z}$ >=$\dfrac{(x^4-x)(y+z+1)}{(x+y+z)^2}$
tương tư với 3 phân thức còn lại
sau đó lam đơn giản thôi

Ông ngước dấu rùi :wacko:



#18
khoacktv

khoacktv

    Lính mới

  • Thành viên
  • 6 Bài viết

chào các bạn. Mình có 1 số vấn đề muốn hỏi. Hiện giờ có 1 ý kiến dựa vào $f(x)\leq m \forall x\Leftrightarrow Max f(x)\leq m$ để CM BDT sau: Cho x,y,z>0 x+y+z=3

P=$\frac{x^{2}}{1+y^{2}}+\frac{y^{2}}{1+z^{2}}+\frac{z^{2}}{1+x^{2}}\geq \frac{3}{2}$

Bài giải

Sử dụng BDT cauchy schwarz ta có P $\geq \frac{(x+y+z)^{2}}{3+x^{2}+y^{2}+z^{2}}$=f(x,y,z)  $\forall$ x,y,z>0

Mà f(x,y,z)$\leq \frac{(x+y+z)^{2}}{3+\frac{(x+y+z)^{2}}{3}}$=3/2 $\forall$ x,y,z>0

vì $P\geq f(x,y,z)\Leftrightarrow P\geq Max f(x,y,z)=3/2 hay P\geq 3/2 (đpcm)$

Bạn xem bài chứng minh này đúng không. Nếu sai mong bạn nói rõ chỗ sai dùm.



#19
hoanganhhaha

hoanganhhaha

    Trung sĩ

  • Thành viên
  • 131 Bài viết

$P=\sum \frac{x^2(1+y^2)-x^2y^2}{1+y^2}\geq x^2-\frac{x^2y^2}{2y}=\sum x^2-\sum \frac{x^2y}{2}$

 

mà ta có$(x+y+z)(x^2+y^2+z^2)=(x^3+xy^2)+(y^3+yz^2)+(z^3+zx^2)+(x^2y+y^2z+z^2x)\geq 3(x^2y+y^2z+z^2x)$

 

do đó$\sum x^2 -\frac{1}{2}\sum x^2y\geq \sum x^2- \frac{1}{2}\sum x^2$= $\frac{1}{2}\sum x^2\geq \frac{3}{2}$



#20
I Love MC

I Love MC

    Đại úy

  • Thành viên nổi bật 2016
  • 1861 Bài viết

Mình chế được cái bày này ( không biết đụng hàng ai chưa ) 
Cho $\sqrt{\frac{x}{y+z}}=a$ $\sqrt{\frac{y}{x+z}}=b$ $\sqrt{\frac{z}{x+y}}=c$ với $x,y,z>0$ 
C/m $a.b+bc+ac>2,5$


Bài viết đã được chỉnh sửa nội dung bởi I Love MC: 07-06-2014 - 12:33





2 người đang xem chủ đề

0 thành viên, 2 khách, 0 thành viên ẩn danh